Happy Memorial Day! Please note that AoPS Online is closed May 24-26th.

G
Topic
First Poster
Last Poster
side ratio in triangle, angle bisectors form cyclic quadrilateral
jasperE3   3
N 43 minutes ago by lpieleanu
Source: Mongolia MO 2000 Grade 10 P6
In a triangle $ABC$, the angle bisector at $A,B,C$ meet the opposite sides at $A_1,B_1,C_1$, respectively. Prove that if the quadrilateral $BA_1B_1C_1$ is cyclic, then
$$\frac{AC}{AB+BC}=\frac{AB}{AC+CB}+\frac{BC}{BA+AC}.$$
3 replies
jasperE3
Apr 22, 2021
lpieleanu
43 minutes ago
Serbian selection contest for the IMO 2025 - P6
OgnjenTesic   5
N an hour ago by JARP091
Source: Serbian selection contest for the IMO 2025
For an $n \times n$ table filled with natural numbers, we say it is a divisor table if:
- the numbers in the $i$-th row are exactly all the divisors of some natural number $r_i$,
- the numbers in the $j$-th column are exactly all the divisors of some natural number $c_j$,
- $r_i \ne r_j$ for every $i \ne j$.

A prime number $p$ is given. Determine the smallest natural number $n$, divisible by $p$, such that there exists an $n \times n$ divisor table, or prove that such $n$ does not exist.

Proposed by Pavle Martinović
5 replies
OgnjenTesic
May 22, 2025
JARP091
an hour ago
JBMO TST Bosnia and Herzegovina 2024 P3
FishkoBiH   2
N an hour ago by Ianis
Source: JBMO TST Bosnia and Herzegovina 2024 P3
Let $ABC$ be a right-angled triangle where $ACB$=90°.Let $CD$ be an altitude of that triangle and points $M$ and $N$ be the midpoints of $CD$ and $BC$, respectively.If $S$ is the circumcenter of the triangle $AMN$, prove that $AS$ and $BC$ are paralel.
2 replies
FishkoBiH
5 hours ago
Ianis
an hour ago
A line parallel to the asymptote of a cubic
kosmonauten3114   0
2 hours ago
Source: My own, but uninspiring...
Given a scalene triangle $\triangle{ABC}$, let $P$ be a point ($\neq \text{X(4)}$). Let $P'$ be the anticomplement of $P$, and let $Q$ be the $\text{X(1)}$-anticomplementary conjugate of $P$. Prove that the line $P'Q$ is parallel to the real asymptote of the circular pivotal isocubic with pivot $P$.
0 replies
kosmonauten3114
2 hours ago
0 replies
My Unsolved Problem
ZeltaQN2008   1
N 2 hours ago by Tung-CHL
Source: IDK
Given a positive integer \( m \) and \( a > 1 \). Prove that there always exists a positive integer \( n \) such that \( m \mid (a^n + n) \).

P/s: I can prove the problem if $m$ is a power of a prime number, but for arbitrary $m$ then well.....
1 reply
ZeltaQN2008
Yesterday at 10:18 AM
Tung-CHL
2 hours ago
Please be my guest!
Madunglecha   0
2 hours ago
Find all the (a,b,c) are integers which suggests
a^4-6a^2b^2+b^4=-c^2
0 replies
Madunglecha
2 hours ago
0 replies
sequence (.) eventually becomes constant.
N.T.TUAN   61
N 2 hours ago by BS2012
Source: USAMO 2007
Let $n$ be a positive integer. Define a sequence by setting $a_{1}= n$ and, for each $k > 1$, letting $a_{k}$ be the unique integer in the range $0\leq a_{k}\leq k-1$ for which $a_{1}+a_{2}+...+a_{k}$ is divisible by $k$. For instance, when $n = 9$ the obtained sequence is $9,1,2,0,3,3,3,...$. Prove that for any $n$ the sequence $a_{1},a_{2},...$ eventually becomes constant.
61 replies
N.T.TUAN
Apr 26, 2007
BS2012
2 hours ago
2025 consecutive numbers are divisible by 2026
cuden   0
2 hours ago
Source: Collect
Problem..
0 replies
cuden
2 hours ago
0 replies
Lots of perpendiculars; compute HQ/HR
MellowMelon   55
N 2 hours ago by Stead
Source: USA TST 2011 P1
In an acute scalene triangle $ABC$, points $D,E,F$ lie on sides $BC, CA, AB$, respectively, such that $AD \perp BC, BE \perp CA, CF \perp AB$. Altitudes $AD, BE, CF$ meet at orthocenter $H$. Points $P$ and $Q$ lie on segment $EF$ such that $AP \perp EF$ and $HQ \perp EF$. Lines $DP$ and $QH$ intersect at point $R$. Compute $HQ/HR$.

Proposed by Zuming Feng
55 replies
MellowMelon
Jul 26, 2011
Stead
2 hours ago
Another right angled triangle
ariopro1387   0
3 hours ago
Source: Iran Team selection test 2025 - P7
Let $ABC$ be a right angled triangle with $\angle A=90$. Point $M$ is the midpoint of side $BC$ And $P$ be an arbitrary point on $AM$. The reflection of $BP$ over $AB$ intersects lines $AC$ and $AM$ at $T$ and $Q$, respectively. The circumcircles of $BPQ$ and $ABC$ intersect again at $F$. Prove that the center of the circumcircle of $CFT$ lies on $BQ$.
0 replies
ariopro1387
3 hours ago
0 replies
diophantine equation
m4thbl3nd3r   1
N 3 hours ago by whwlqkd
Find all positive integers $n,k$ such that $$5^{2n+1}-5^n+1=k^2$$
1 reply
m4thbl3nd3r
Today at 10:34 AM
whwlqkd
3 hours ago
A well-known geo configuration revisited
Tintarn   6
N Apr 10, 2025 by Primeniyazidayi
Source: Dutch TST 2024, 2.1
Let $ABC$ be a triangle with orthocenter $H$ and circumcircle $\Gamma$. Let $D$ be the reflection of $A$ in $B$ and let $E$ the reflection of $A$ in $C$. Let $M$ be the midpoint of segment $DE$. Show that the tangent to $\Gamma$ in $A$ is perpendicular to $HM$.
6 replies
Tintarn
Jun 28, 2024
Primeniyazidayi
Apr 10, 2025
A well-known geo configuration revisited
G H J
G H BBookmark kLocked kLocked NReply
Source: Dutch TST 2024, 2.1
The post below has been deleted. Click to close.
This post has been deleted. Click here to see post.
Tintarn
9045 posts
#1 • 1 Y
Y by ehuseyinyigit
Let $ABC$ be a triangle with orthocenter $H$ and circumcircle $\Gamma$. Let $D$ be the reflection of $A$ in $B$ and let $E$ the reflection of $A$ in $C$. Let $M$ be the midpoint of segment $DE$. Show that the tangent to $\Gamma$ in $A$ is perpendicular to $HM$.
Z K Y
The post below has been deleted. Click to close.
This post has been deleted. Click here to see post.
sami1618
913 posts
#2
Y by
Let $N$ be the midpoint of $BC$. Then reflecting about $N$ sends $M$ to $A$ and $H$ to the $A$ antipode, proving the claim.
Z K Y
The post below has been deleted. Click to close.
This post has been deleted. Click here to see post.
YaoAOPS
1541 posts
#3 • 2 Y
Y by sami1618, ehuseyinyigit
$a, b, c$ on unit circle, then $h = a + b + c, d = 2 \cdot b - a, e = 2 \cdot c - a, m = \frac{d+e}{2} = b + c - a, h - m = 2a$.
Z K Y
The post below has been deleted. Click to close.
This post has been deleted. Click here to see post.
lian_the_noob12
173 posts
#4
Y by
Solution by Sir Jawad(TanR314) IMO gold orz orz orz
$I$ be the midpoint of $BC.$ Reflect $H$ over $I$ to $H'$ and Reflecting $A$ over $I$ sends to $M$. Hence $AHMH'$ is a parallelogram.
$$\angle XAH+\angle XHA=\angle XAH +\angle HAH'=90° \blacksquare$$
Another solution is by noting $BHCM$ is the nine-point circle of $\triangle ADC$ then showing $\angle XHA=|\angle D -\angle E|$
This post has been edited 1 time. Last edited by lian_the_noob12, Aug 2, 2024, 4:21 PM
Z K Y
The post below has been deleted. Click to close.
This post has been deleted. Click here to see post.
Z4ADies
64 posts
#5
Y by
idk why i couldn't see the synthetic solution.... :(
Computations left to reader.
Let $N$ be midpoint of $BC$. Take $N$ as the origin. $N=(0,0), B=(-1,0), C=(1,0), A=(a,b)$. From $NA=NM$ $\implies$ $M=(-a,-b)$. From $AO=BO$ $\implies$ $O=(0,\frac{a^2+b^2-1}{2b})$. Take altitudes from $B$ and $C$ to $AC$ and $AB$ respectively and get equations of lines and their intersection which is $H$. So, $H=(a,\frac{a^2-1}{-b})$. We need to prove $AO$ is parallel to $HM$. Which means their slopes should be equal.
Which is true since, $\frac{b-\frac{a^2+b^2-1}{2b}}{a}=\frac{\frac{a^2-1}{-b}+b}{2a}$.
Z K Y
The post below has been deleted. Click to close.
This post has been deleted. Click here to see post.
iced_tea
4 posts
#6
Y by
Let $O$ and $G$ be the circumcentre and centroid of $\triangle ABC$ respectively.
Note that the homothety with centre G and ratio $-\dfrac{1}{2}$ maps $H \mapsto O, M \mapsto A$ so $HM \parallel AO$. $\blacksquare$
This post has been edited 1 time. Last edited by iced_tea, Mar 26, 2025, 1:08 PM
Reason: typo
Z K Y
The post below has been deleted. Click to close.
This post has been deleted. Click here to see post.
Primeniyazidayi
112 posts
#7
Y by
I'm going to use barycentric coordinates.
Let $A=(1,0,0)$,$B=(0,1,0)$,$C=(0,0,1)$;$AB=c$,$BC=a$,$CA=b$.Then $H=(\frac{1}{S_A}:\frac{1}{S_B}:\frac{1}{S_C})$ and $M=(-1,1,1)$.Note that the displacement vectors can be written as $\vec{MH}=(\frac{S^2+S_{BC}}{S^2},\frac{S_{AC}-S^2}{S^2},\frac{S_{AB}-S^2}{S^2})$ and $\vec{AA}=(b^2-c^2,-b^2,c^2)$.Then,by EFFT,if we have that $$a^2[c^2\frac{S_{AC}-S^2}{S^2}-b^2\frac{S_{AB}-S^2}{S^2}]+b^2[(b^2-c^2)\frac{S_{AB}-S^2}{S^2}+c^2\frac{S^2+S_{BC}}{S^2}]+c^2[(b^2-c^2)\frac{S_{AC}-S^2}{S^2}-b^2\frac{S^2+S_{BC}}{S^2}]=0,$$we are done. This can be shown by some algebraic manipulations.
This post has been edited 2 times. Last edited by Primeniyazidayi, Apr 11, 2025, 5:24 AM
Z K Y
N Quick Reply
G
H
=
a